¿Cómo responder a excepción preguntas de argumentos en el LSAT

Muchos fortalecer / debilitar preguntas sobre el LSAT no piden la opción que mejor se fortalece o debilita (o soportes, o socava) el argumento pero en lugar de la opción que ¿no hacer una de esas cosas. Estas preguntas terminan en la palabra de suma importancia EXCEPCIÓN. Siempre ha capitalizado, por lo que no se lo puede perder.

Estas preguntas tienen este aspecto:

  • Cada uno de los siguientes, si es cierto, es compatible con el argumento anterior, excepto:

  • Cada uno de los siguientes, si es cierto, debilitaría el argumento de que el comentarista EXCEPTO:

  • Cada uno de los siguientes, si es cierto, apoya la hipótesis de que el físico EXCEPTO:

  • Cada uno de los siguientes, si es cierto, debilita el argumento EXCEPTO:

  • Cada uno de los siguientes, si es cierto, refuerza el argumento EXCEPTO:

También pueden tener este aspecto:

  • ¿Cuál de los siguientes, si todos ellos son verdaderas, es MENOS útil para establecer que la conclusión anterior se dibuja correctamente?

  • ¿Cuál de los siguientes, si todos ellos son verdaderas, es MENOS efectiva a socavar el argumento de los políticos?

Todavía están fortalecen / debilitan preguntas, pero en lugar de encontrar una respuesta para fortalecer / debilitar / support / socavar te quieres encontrar cuatro.

Como siempre, lea primero la pregunta cuidadosamente. Estas preguntas no son el estándar de fortalecer / debilitar preguntas. En su lugar, ofrecen cuatro opciones que fortalecen o debilitan la conclusión, y uno que no lo hace. No necesariamente tiene que encontrar una respuesta que fortalece o debilita la conclusión- lo que desea es una respuesta que no hace lo que los otros cuatro lo hacen.

Por ejemplo, si la pregunta lee " Cada uno de los siguientes, si es cierto, debilita el argumento EXCEPTO: " la respuesta correcta no refuerza necesariamente el argumento, pero definitivamente no debilitarlo.

He aquí un ejemplo de una pregunta que tiene cuatro respuestas que debilitan el argumento y una que no lo hace:

Algunos trabajadores en las fábricas de palomitas de maíz de microondas han contraído una enfermedad pulmonar rara. Los expertos han vinculado esta enfermedad a un producto químico utilizado en el proceso de mezcla de palomitas de maíz y saborizantes. Los consumidores deben, por lo tanto, dejar de comprar y comer todo tipo de palomitas de microondas para evitar el riesgo de contraer esta enfermedad pulmonar.

Cada uno de los siguientes, si es cierto, debilita el argumento EXCEPTO:

  • (A) La enfermedad pulmonar es causado por la exposición a la sustancia química durante muchas horas a la vez durante un período de años.

  • (B) El producto químico sólo se convierte en tóxico cuando se mantiene a una temperatura mucho mayor que la alcanzada por las palomitas de maíz en un microondas.

  • (C) La enfermedad pulmonar sólo se ha encontrado en los trabajadores que manejan el sabor especia de Cajun de palomitas de maíz.

  • (D) En 20 años de extendido consumo de palomitas de microondas, ningún consumidor nunca ha contraído esta enfermedad pulmonar rara.

  • (E) La EPA aún no ha hecho ninguna investigación para determinar si el producto químico que causa la enfermedad pulmonar está presente en el vapor y el aire que sale de una bolsa de palomitas de maíz cuando se abre.

De la lectura de la pregunta primero, usted sabe que quiere una respuesta que no debilita el argumento. La conclusión es que los consumidores deben dejar de comer todo tipo de palomitas de microondas para evitar contraer una enfermedad. La evidencia de esta conclusión es que un producto químico en palomitas de microondas causó esta enfermedad en empleados de la fábrica de palomitas de maíz.

Lo que debilita el argumento? Cualquier evidencia que hace que los consumidores parecen seguros. Si usted puede demostrar que la enfermedad es de alguna manera restringido a los trabajadores de la fábrica, puede tranquilizar a los consumidores de que pueden comer las palomitas de microondas con impunidad. Lo que no debilita la conclusión? Cualquier evidencia que indica que los consumidores pueden contraer la enfermedad por comer palomitas de microondas, o cualquier evidencia de que es de relevancia clara o simplemente fuera de tema.

  • Choice (A) debilita el argumento porque la mayoría de los consumidores no están expuestos a la sustancia química en cuestión durante largos períodos de tiempo.

  • La opción (B) se debilita el argumento porque los consumidores no están manteniendo sus palomitas suficientemente caliente.

  • La opción (C) debilita la sugerencia de que los consumidores deben evitar todo tipo de palomitas de microondas, aunque posiblemente no la sugerencia de que se debe evitar la especia de Cajun sabor.

  • Choice (D) debilita la sugerencia porque hace que el riesgo para los consumidores parecen casi insignificante.

  • La opción (E) no debilita el argumento: si el EPA aún no ha hecho ninguna investigación sobre la presencia de la química en el vapor de palomitas de maíz, entonces la química bien podría ser que acechan ahí, listo para enfermar a los consumidores desventurados.

La opción (E) no especialmente a reforzar el argumento, pero no debilita, tampoco. La opción (E) es la respuesta correcta.




» » » » ¿Cómo responder a excepción preguntas de argumentos en el LSAT